Finding the radius of convergence for $frac{1}{(3 - x)(1 + x)^{2}}$ at $x_{0} = 0$ and $x_{0} = 1$











up vote
2
down vote

favorite
1












I am having trouble with the problem




Find the radius of convergence for $f(x) = frac{1}{(3 - x)(1 +
x)^{2}}$
at $x_{0} = 0$ and $x_{1} = 1$.




Usually, when I'm trying to find radius of convergence, I'm given some sort of series (like, a summation). This isn't the case here, and I'm confused as to what I'm supposed to do. This is a problem in the exercise. It is in the section for power series. Also, I have this formula for the radius of convergence $R$:



$$R = frac{1}{limsup_{ntoinfty} |a_{n}|^{1/n}},$$



but I'm not sure how I'm supposed to use that here either.



Any help is appreciated.



EDIT: Maybe it has something to do with the standard Taylor series form? Like,



$$f(x) = f(x_{0}) + f'(x_{0})(x - x_{0}) + f''(x_{0})(x - x_{0})^{2}/2! ldots $$



Also, in case it matters, by Partial Fractions, you can write



$$frac{1}{(3-x)(1 + x)^{2}} = frac{1}{16(x + 1)} + frac{1}{4(x+1)^{2}} - frac{1}{16(x - 3)}. $$



The first few terms of the Taylor series expansion about $x_{0} = 0$ is given by



$$f(x) = frac{1}{3} - frac{5}{9}x + frac{22}{27}x^{2} - frac{86}{81}x^{3} + frac{319}{243}x^{4} + cdots $$



(I don't have a closed term for the sum, but these are the first terms explicitly computed. Looks like the denominator is just powers of $3$. Not sure about the numerator though...)










share|cite|improve this question
























  • The radius of convergence is the distance to the nearest pole (place where the denominator vanishes, when considered as a function of a complex variable).
    – Ethan Bolker
    Nov 12 at 17:07












  • hi @Ethan Bolker, I don't know what a pole is. this is in a real analysis book. i think poles are from complex analysis
    – joseph
    Nov 12 at 17:08












  • Poles are from complex analysis but they influence real power series. The series for $1/(1+x^2)$ converges on $(-1, 1)$ because the denominator vanishes at $x=i$. Perhaps you're not allowed to know this yet.
    – Ethan Bolker
    Nov 12 at 17:10












  • I guess not, but it kind of makes sense. A pole is like an asymptote, I think? Do you know about any other ways that this problem can be solved?
    – joseph
    Nov 12 at 17:14












  • Well in your examples the poles are real. The power series surely couldn't converge to the function there, so the distance to the pole is an upper bound on the radius of convergence. I think you will have to wait now for someone to provide a complete answer.
    – Ethan Bolker
    Nov 12 at 17:19















up vote
2
down vote

favorite
1












I am having trouble with the problem




Find the radius of convergence for $f(x) = frac{1}{(3 - x)(1 +
x)^{2}}$
at $x_{0} = 0$ and $x_{1} = 1$.




Usually, when I'm trying to find radius of convergence, I'm given some sort of series (like, a summation). This isn't the case here, and I'm confused as to what I'm supposed to do. This is a problem in the exercise. It is in the section for power series. Also, I have this formula for the radius of convergence $R$:



$$R = frac{1}{limsup_{ntoinfty} |a_{n}|^{1/n}},$$



but I'm not sure how I'm supposed to use that here either.



Any help is appreciated.



EDIT: Maybe it has something to do with the standard Taylor series form? Like,



$$f(x) = f(x_{0}) + f'(x_{0})(x - x_{0}) + f''(x_{0})(x - x_{0})^{2}/2! ldots $$



Also, in case it matters, by Partial Fractions, you can write



$$frac{1}{(3-x)(1 + x)^{2}} = frac{1}{16(x + 1)} + frac{1}{4(x+1)^{2}} - frac{1}{16(x - 3)}. $$



The first few terms of the Taylor series expansion about $x_{0} = 0$ is given by



$$f(x) = frac{1}{3} - frac{5}{9}x + frac{22}{27}x^{2} - frac{86}{81}x^{3} + frac{319}{243}x^{4} + cdots $$



(I don't have a closed term for the sum, but these are the first terms explicitly computed. Looks like the denominator is just powers of $3$. Not sure about the numerator though...)










share|cite|improve this question
























  • The radius of convergence is the distance to the nearest pole (place where the denominator vanishes, when considered as a function of a complex variable).
    – Ethan Bolker
    Nov 12 at 17:07












  • hi @Ethan Bolker, I don't know what a pole is. this is in a real analysis book. i think poles are from complex analysis
    – joseph
    Nov 12 at 17:08












  • Poles are from complex analysis but they influence real power series. The series for $1/(1+x^2)$ converges on $(-1, 1)$ because the denominator vanishes at $x=i$. Perhaps you're not allowed to know this yet.
    – Ethan Bolker
    Nov 12 at 17:10












  • I guess not, but it kind of makes sense. A pole is like an asymptote, I think? Do you know about any other ways that this problem can be solved?
    – joseph
    Nov 12 at 17:14












  • Well in your examples the poles are real. The power series surely couldn't converge to the function there, so the distance to the pole is an upper bound on the radius of convergence. I think you will have to wait now for someone to provide a complete answer.
    – Ethan Bolker
    Nov 12 at 17:19













up vote
2
down vote

favorite
1









up vote
2
down vote

favorite
1






1





I am having trouble with the problem




Find the radius of convergence for $f(x) = frac{1}{(3 - x)(1 +
x)^{2}}$
at $x_{0} = 0$ and $x_{1} = 1$.




Usually, when I'm trying to find radius of convergence, I'm given some sort of series (like, a summation). This isn't the case here, and I'm confused as to what I'm supposed to do. This is a problem in the exercise. It is in the section for power series. Also, I have this formula for the radius of convergence $R$:



$$R = frac{1}{limsup_{ntoinfty} |a_{n}|^{1/n}},$$



but I'm not sure how I'm supposed to use that here either.



Any help is appreciated.



EDIT: Maybe it has something to do with the standard Taylor series form? Like,



$$f(x) = f(x_{0}) + f'(x_{0})(x - x_{0}) + f''(x_{0})(x - x_{0})^{2}/2! ldots $$



Also, in case it matters, by Partial Fractions, you can write



$$frac{1}{(3-x)(1 + x)^{2}} = frac{1}{16(x + 1)} + frac{1}{4(x+1)^{2}} - frac{1}{16(x - 3)}. $$



The first few terms of the Taylor series expansion about $x_{0} = 0$ is given by



$$f(x) = frac{1}{3} - frac{5}{9}x + frac{22}{27}x^{2} - frac{86}{81}x^{3} + frac{319}{243}x^{4} + cdots $$



(I don't have a closed term for the sum, but these are the first terms explicitly computed. Looks like the denominator is just powers of $3$. Not sure about the numerator though...)










share|cite|improve this question















I am having trouble with the problem




Find the radius of convergence for $f(x) = frac{1}{(3 - x)(1 +
x)^{2}}$
at $x_{0} = 0$ and $x_{1} = 1$.




Usually, when I'm trying to find radius of convergence, I'm given some sort of series (like, a summation). This isn't the case here, and I'm confused as to what I'm supposed to do. This is a problem in the exercise. It is in the section for power series. Also, I have this formula for the radius of convergence $R$:



$$R = frac{1}{limsup_{ntoinfty} |a_{n}|^{1/n}},$$



but I'm not sure how I'm supposed to use that here either.



Any help is appreciated.



EDIT: Maybe it has something to do with the standard Taylor series form? Like,



$$f(x) = f(x_{0}) + f'(x_{0})(x - x_{0}) + f''(x_{0})(x - x_{0})^{2}/2! ldots $$



Also, in case it matters, by Partial Fractions, you can write



$$frac{1}{(3-x)(1 + x)^{2}} = frac{1}{16(x + 1)} + frac{1}{4(x+1)^{2}} - frac{1}{16(x - 3)}. $$



The first few terms of the Taylor series expansion about $x_{0} = 0$ is given by



$$f(x) = frac{1}{3} - frac{5}{9}x + frac{22}{27}x^{2} - frac{86}{81}x^{3} + frac{319}{243}x^{4} + cdots $$



(I don't have a closed term for the sum, but these are the first terms explicitly computed. Looks like the denominator is just powers of $3$. Not sure about the numerator though...)







calculus real-analysis power-series taylor-expansion






share|cite|improve this question















share|cite|improve this question













share|cite|improve this question




share|cite|improve this question








edited Nov 12 at 17:32

























asked Nov 12 at 17:05









joseph

385




385












  • The radius of convergence is the distance to the nearest pole (place where the denominator vanishes, when considered as a function of a complex variable).
    – Ethan Bolker
    Nov 12 at 17:07












  • hi @Ethan Bolker, I don't know what a pole is. this is in a real analysis book. i think poles are from complex analysis
    – joseph
    Nov 12 at 17:08












  • Poles are from complex analysis but they influence real power series. The series for $1/(1+x^2)$ converges on $(-1, 1)$ because the denominator vanishes at $x=i$. Perhaps you're not allowed to know this yet.
    – Ethan Bolker
    Nov 12 at 17:10












  • I guess not, but it kind of makes sense. A pole is like an asymptote, I think? Do you know about any other ways that this problem can be solved?
    – joseph
    Nov 12 at 17:14












  • Well in your examples the poles are real. The power series surely couldn't converge to the function there, so the distance to the pole is an upper bound on the radius of convergence. I think you will have to wait now for someone to provide a complete answer.
    – Ethan Bolker
    Nov 12 at 17:19


















  • The radius of convergence is the distance to the nearest pole (place where the denominator vanishes, when considered as a function of a complex variable).
    – Ethan Bolker
    Nov 12 at 17:07












  • hi @Ethan Bolker, I don't know what a pole is. this is in a real analysis book. i think poles are from complex analysis
    – joseph
    Nov 12 at 17:08












  • Poles are from complex analysis but they influence real power series. The series for $1/(1+x^2)$ converges on $(-1, 1)$ because the denominator vanishes at $x=i$. Perhaps you're not allowed to know this yet.
    – Ethan Bolker
    Nov 12 at 17:10












  • I guess not, but it kind of makes sense. A pole is like an asymptote, I think? Do you know about any other ways that this problem can be solved?
    – joseph
    Nov 12 at 17:14












  • Well in your examples the poles are real. The power series surely couldn't converge to the function there, so the distance to the pole is an upper bound on the radius of convergence. I think you will have to wait now for someone to provide a complete answer.
    – Ethan Bolker
    Nov 12 at 17:19
















The radius of convergence is the distance to the nearest pole (place where the denominator vanishes, when considered as a function of a complex variable).
– Ethan Bolker
Nov 12 at 17:07






The radius of convergence is the distance to the nearest pole (place where the denominator vanishes, when considered as a function of a complex variable).
– Ethan Bolker
Nov 12 at 17:07














hi @Ethan Bolker, I don't know what a pole is. this is in a real analysis book. i think poles are from complex analysis
– joseph
Nov 12 at 17:08






hi @Ethan Bolker, I don't know what a pole is. this is in a real analysis book. i think poles are from complex analysis
– joseph
Nov 12 at 17:08














Poles are from complex analysis but they influence real power series. The series for $1/(1+x^2)$ converges on $(-1, 1)$ because the denominator vanishes at $x=i$. Perhaps you're not allowed to know this yet.
– Ethan Bolker
Nov 12 at 17:10






Poles are from complex analysis but they influence real power series. The series for $1/(1+x^2)$ converges on $(-1, 1)$ because the denominator vanishes at $x=i$. Perhaps you're not allowed to know this yet.
– Ethan Bolker
Nov 12 at 17:10














I guess not, but it kind of makes sense. A pole is like an asymptote, I think? Do you know about any other ways that this problem can be solved?
– joseph
Nov 12 at 17:14






I guess not, but it kind of makes sense. A pole is like an asymptote, I think? Do you know about any other ways that this problem can be solved?
– joseph
Nov 12 at 17:14














Well in your examples the poles are real. The power series surely couldn't converge to the function there, so the distance to the pole is an upper bound on the radius of convergence. I think you will have to wait now for someone to provide a complete answer.
– Ethan Bolker
Nov 12 at 17:19




Well in your examples the poles are real. The power series surely couldn't converge to the function there, so the distance to the pole is an upper bound on the radius of convergence. I think you will have to wait now for someone to provide a complete answer.
– Ethan Bolker
Nov 12 at 17:19










3 Answers
3






active

oldest

votes

















up vote
1
down vote



accepted










First, write the given series as a power one around some given point, then evaluate the radius of convergence.



For example, about $;x=1;$ :



$$frac1{(3-x)(x+1)^2}=-frac1{16}frac1{x-3}+frac1{16}frac1{x+1}+frac14frac1{(x+1)^2}=$$



$$=frac1{32}frac1{1-frac{x-1}2}+frac1{32}frac1{1+frac{x-1}2}+frac1{16}frac1{left(1+frac{x-1}2right)^2}=$$$${}$$



$$=frac1{32}left[sum_{n=0}^infty(-1)^nfrac{(x-1)^n}{2^n}+sum_{n=0}^inftyfrac{(x-1)^n}{2^n}+2left(sum_{n=1}^infty(-1)^nfrac{(x-1)^n}{2^n}right)^2right]$$



Now, if you had to write down a power series then that'd be really tedious, though not quite difficult, but you're only required to calculate the convergence radius, and that seems pretty easy as all the three series' convergence radius above is $;1;$ around the point we want, so it must be



$$left|frac{x-1}2right|<1iff |x-1|<2$$



Note that in fact all we needed above is what follows after the simple fractions decomposition, as we already there have the basic $;frac{x-1}2;$ thing we shall need. To write down the infinite series is just for fun...






share|cite|improve this answer























  • Check carefully, in case you want/need to do it, the signs of the simple fractions as I have some minor doubts about that...but that really is irrelevant for the direct question you've asked.
    – DonAntonio
    Nov 12 at 17:44










  • In the first summation term, why is it $(x - 1)^{2}$ rather than $(x - 1)^{n}$?
    – joseph
    Nov 12 at 18:05










  • @joseph Simple typo. Corrected.
    – DonAntonio
    Nov 12 at 18:18










  • I think your partial fraction decomposition in the first equation is incorrect. The coefficient of $frac{1}{4}$ should be $+$ not $-$.
    – joseph
    Nov 12 at 19:23










  • @joseph It really doesn't matter, as already written there. You can do it correctly. The important point is already there...
    – DonAntonio
    Nov 12 at 19:33


















up vote
1
down vote













If you represent $frac {1}{(3-x)(1+x)^2}$ as a Taylor series, centered at the points identified above that Taylor series will have a finite radius. There is enough information presented in the question to find these radii without deriving the entire series.



However, if you represent the function as a Laurent series, you have more choices in how you represent the series and depending on those choices the series will converge in different annuli.



The trick comes down to finding the discontinuities of the function. Since the Taylor series is continuous, it is going to diverge as these discontinuities approach.






share|cite|improve this answer























  • is this a good way to do it? the taylor series representation of the function looks very ugly wolframalpha.com/input/…
    – joseph
    Nov 12 at 17:33












  • If you work out the Taylor series centered at $0,$ you are going to get something of the form $sum A (frac {x}{3})^n + B (-1)^nx^n + Cn(-1)^n x^n$
    – Doug M
    Nov 12 at 17:47




















up vote
1
down vote













Here are my thoughts. This might not be the original purpose of the exercise, but this is my guess.



According to your partial fraction decomposition, and using the well-know power series $(1-x)^{-1} = sum_0^infty x^n$ for $x in (-1,1)$, at $x=0$, we have
begin{align*}
f(x)&= frac 1{16}sum_0^infty (-1)^n x^n - frac {mathrm d}{mathrm dx} left( frac 1{4(x+1)} right) + frac 1{48} cdot frac 1{1 - x/3} \
&= frac 1{16}sum_0^infty (-1)^n x^n - frac 14 cdotfrac {mathrm d }{mathrm dx}left( sum_0^infty (-1)^n x^n right) + frac 1{48} cdot sum_0^infty frac {x^n}{3^n}\
&= frac 1{16}sum_0^infty (-1)^n x^n - frac 14 sum_0^infty (n+1)(-1)^{n+1} x^{n} + frac 1{48} cdot sum_0^infty frac {x^n}{3^n}\
&= sum_0^infty left(frac {(-1)^n}{16} + frac 14 (n+1)(-1)^{n+1} +frac 1{16 cdot 3^{n+1}} right) x^n,
end{align*}

and all of these computations are valid iff $|x|<1$ and $|x/3|<1$. Conclusively the radius is $1$. You might try to compute again by the formula.



At $x=1$, just let $y = x-1$, then $x = y+1$ and
$$
f(x) =g(y) = frac 1{(2-y)(2+y)^2},
$$

you could do the similar thing like above to give the power series at $y=0$. You could see that the operations are valid iff $|y/2|<1$, hence $|y|<2$ and the radius is $2$.






share|cite|improve this answer





















    Your Answer





    StackExchange.ifUsing("editor", function () {
    return StackExchange.using("mathjaxEditing", function () {
    StackExchange.MarkdownEditor.creationCallbacks.add(function (editor, postfix) {
    StackExchange.mathjaxEditing.prepareWmdForMathJax(editor, postfix, [["$", "$"], ["\\(","\\)"]]);
    });
    });
    }, "mathjax-editing");

    StackExchange.ready(function() {
    var channelOptions = {
    tags: "".split(" "),
    id: "69"
    };
    initTagRenderer("".split(" "), "".split(" "), channelOptions);

    StackExchange.using("externalEditor", function() {
    // Have to fire editor after snippets, if snippets enabled
    if (StackExchange.settings.snippets.snippetsEnabled) {
    StackExchange.using("snippets", function() {
    createEditor();
    });
    }
    else {
    createEditor();
    }
    });

    function createEditor() {
    StackExchange.prepareEditor({
    heartbeatType: 'answer',
    convertImagesToLinks: true,
    noModals: true,
    showLowRepImageUploadWarning: true,
    reputationToPostImages: 10,
    bindNavPrevention: true,
    postfix: "",
    imageUploader: {
    brandingHtml: "Powered by u003ca class="icon-imgur-white" href="https://imgur.com/"u003eu003c/au003e",
    contentPolicyHtml: "User contributions licensed under u003ca href="https://creativecommons.org/licenses/by-sa/3.0/"u003ecc by-sa 3.0 with attribution requiredu003c/au003e u003ca href="https://stackoverflow.com/legal/content-policy"u003e(content policy)u003c/au003e",
    allowUrls: true
    },
    noCode: true, onDemand: true,
    discardSelector: ".discard-answer"
    ,immediatelyShowMarkdownHelp:true
    });


    }
    });














     

    draft saved


    draft discarded


















    StackExchange.ready(
    function () {
    StackExchange.openid.initPostLogin('.new-post-login', 'https%3a%2f%2fmath.stackexchange.com%2fquestions%2f2995568%2ffinding-the-radius-of-convergence-for-frac13-x1-x2-at-x-0%23new-answer', 'question_page');
    }
    );

    Post as a guest















    Required, but never shown

























    3 Answers
    3






    active

    oldest

    votes








    3 Answers
    3






    active

    oldest

    votes









    active

    oldest

    votes






    active

    oldest

    votes








    up vote
    1
    down vote



    accepted










    First, write the given series as a power one around some given point, then evaluate the radius of convergence.



    For example, about $;x=1;$ :



    $$frac1{(3-x)(x+1)^2}=-frac1{16}frac1{x-3}+frac1{16}frac1{x+1}+frac14frac1{(x+1)^2}=$$



    $$=frac1{32}frac1{1-frac{x-1}2}+frac1{32}frac1{1+frac{x-1}2}+frac1{16}frac1{left(1+frac{x-1}2right)^2}=$$$${}$$



    $$=frac1{32}left[sum_{n=0}^infty(-1)^nfrac{(x-1)^n}{2^n}+sum_{n=0}^inftyfrac{(x-1)^n}{2^n}+2left(sum_{n=1}^infty(-1)^nfrac{(x-1)^n}{2^n}right)^2right]$$



    Now, if you had to write down a power series then that'd be really tedious, though not quite difficult, but you're only required to calculate the convergence radius, and that seems pretty easy as all the three series' convergence radius above is $;1;$ around the point we want, so it must be



    $$left|frac{x-1}2right|<1iff |x-1|<2$$



    Note that in fact all we needed above is what follows after the simple fractions decomposition, as we already there have the basic $;frac{x-1}2;$ thing we shall need. To write down the infinite series is just for fun...






    share|cite|improve this answer























    • Check carefully, in case you want/need to do it, the signs of the simple fractions as I have some minor doubts about that...but that really is irrelevant for the direct question you've asked.
      – DonAntonio
      Nov 12 at 17:44










    • In the first summation term, why is it $(x - 1)^{2}$ rather than $(x - 1)^{n}$?
      – joseph
      Nov 12 at 18:05










    • @joseph Simple typo. Corrected.
      – DonAntonio
      Nov 12 at 18:18










    • I think your partial fraction decomposition in the first equation is incorrect. The coefficient of $frac{1}{4}$ should be $+$ not $-$.
      – joseph
      Nov 12 at 19:23










    • @joseph It really doesn't matter, as already written there. You can do it correctly. The important point is already there...
      – DonAntonio
      Nov 12 at 19:33















    up vote
    1
    down vote



    accepted










    First, write the given series as a power one around some given point, then evaluate the radius of convergence.



    For example, about $;x=1;$ :



    $$frac1{(3-x)(x+1)^2}=-frac1{16}frac1{x-3}+frac1{16}frac1{x+1}+frac14frac1{(x+1)^2}=$$



    $$=frac1{32}frac1{1-frac{x-1}2}+frac1{32}frac1{1+frac{x-1}2}+frac1{16}frac1{left(1+frac{x-1}2right)^2}=$$$${}$$



    $$=frac1{32}left[sum_{n=0}^infty(-1)^nfrac{(x-1)^n}{2^n}+sum_{n=0}^inftyfrac{(x-1)^n}{2^n}+2left(sum_{n=1}^infty(-1)^nfrac{(x-1)^n}{2^n}right)^2right]$$



    Now, if you had to write down a power series then that'd be really tedious, though not quite difficult, but you're only required to calculate the convergence radius, and that seems pretty easy as all the three series' convergence radius above is $;1;$ around the point we want, so it must be



    $$left|frac{x-1}2right|<1iff |x-1|<2$$



    Note that in fact all we needed above is what follows after the simple fractions decomposition, as we already there have the basic $;frac{x-1}2;$ thing we shall need. To write down the infinite series is just for fun...






    share|cite|improve this answer























    • Check carefully, in case you want/need to do it, the signs of the simple fractions as I have some minor doubts about that...but that really is irrelevant for the direct question you've asked.
      – DonAntonio
      Nov 12 at 17:44










    • In the first summation term, why is it $(x - 1)^{2}$ rather than $(x - 1)^{n}$?
      – joseph
      Nov 12 at 18:05










    • @joseph Simple typo. Corrected.
      – DonAntonio
      Nov 12 at 18:18










    • I think your partial fraction decomposition in the first equation is incorrect. The coefficient of $frac{1}{4}$ should be $+$ not $-$.
      – joseph
      Nov 12 at 19:23










    • @joseph It really doesn't matter, as already written there. You can do it correctly. The important point is already there...
      – DonAntonio
      Nov 12 at 19:33













    up vote
    1
    down vote



    accepted







    up vote
    1
    down vote



    accepted






    First, write the given series as a power one around some given point, then evaluate the radius of convergence.



    For example, about $;x=1;$ :



    $$frac1{(3-x)(x+1)^2}=-frac1{16}frac1{x-3}+frac1{16}frac1{x+1}+frac14frac1{(x+1)^2}=$$



    $$=frac1{32}frac1{1-frac{x-1}2}+frac1{32}frac1{1+frac{x-1}2}+frac1{16}frac1{left(1+frac{x-1}2right)^2}=$$$${}$$



    $$=frac1{32}left[sum_{n=0}^infty(-1)^nfrac{(x-1)^n}{2^n}+sum_{n=0}^inftyfrac{(x-1)^n}{2^n}+2left(sum_{n=1}^infty(-1)^nfrac{(x-1)^n}{2^n}right)^2right]$$



    Now, if you had to write down a power series then that'd be really tedious, though not quite difficult, but you're only required to calculate the convergence radius, and that seems pretty easy as all the three series' convergence radius above is $;1;$ around the point we want, so it must be



    $$left|frac{x-1}2right|<1iff |x-1|<2$$



    Note that in fact all we needed above is what follows after the simple fractions decomposition, as we already there have the basic $;frac{x-1}2;$ thing we shall need. To write down the infinite series is just for fun...






    share|cite|improve this answer














    First, write the given series as a power one around some given point, then evaluate the radius of convergence.



    For example, about $;x=1;$ :



    $$frac1{(3-x)(x+1)^2}=-frac1{16}frac1{x-3}+frac1{16}frac1{x+1}+frac14frac1{(x+1)^2}=$$



    $$=frac1{32}frac1{1-frac{x-1}2}+frac1{32}frac1{1+frac{x-1}2}+frac1{16}frac1{left(1+frac{x-1}2right)^2}=$$$${}$$



    $$=frac1{32}left[sum_{n=0}^infty(-1)^nfrac{(x-1)^n}{2^n}+sum_{n=0}^inftyfrac{(x-1)^n}{2^n}+2left(sum_{n=1}^infty(-1)^nfrac{(x-1)^n}{2^n}right)^2right]$$



    Now, if you had to write down a power series then that'd be really tedious, though not quite difficult, but you're only required to calculate the convergence radius, and that seems pretty easy as all the three series' convergence radius above is $;1;$ around the point we want, so it must be



    $$left|frac{x-1}2right|<1iff |x-1|<2$$



    Note that in fact all we needed above is what follows after the simple fractions decomposition, as we already there have the basic $;frac{x-1}2;$ thing we shall need. To write down the infinite series is just for fun...







    share|cite|improve this answer














    share|cite|improve this answer



    share|cite|improve this answer








    edited Nov 12 at 18:18

























    answered Nov 12 at 17:40









    DonAntonio

    175k1491224




    175k1491224












    • Check carefully, in case you want/need to do it, the signs of the simple fractions as I have some minor doubts about that...but that really is irrelevant for the direct question you've asked.
      – DonAntonio
      Nov 12 at 17:44










    • In the first summation term, why is it $(x - 1)^{2}$ rather than $(x - 1)^{n}$?
      – joseph
      Nov 12 at 18:05










    • @joseph Simple typo. Corrected.
      – DonAntonio
      Nov 12 at 18:18










    • I think your partial fraction decomposition in the first equation is incorrect. The coefficient of $frac{1}{4}$ should be $+$ not $-$.
      – joseph
      Nov 12 at 19:23










    • @joseph It really doesn't matter, as already written there. You can do it correctly. The important point is already there...
      – DonAntonio
      Nov 12 at 19:33


















    • Check carefully, in case you want/need to do it, the signs of the simple fractions as I have some minor doubts about that...but that really is irrelevant for the direct question you've asked.
      – DonAntonio
      Nov 12 at 17:44










    • In the first summation term, why is it $(x - 1)^{2}$ rather than $(x - 1)^{n}$?
      – joseph
      Nov 12 at 18:05










    • @joseph Simple typo. Corrected.
      – DonAntonio
      Nov 12 at 18:18










    • I think your partial fraction decomposition in the first equation is incorrect. The coefficient of $frac{1}{4}$ should be $+$ not $-$.
      – joseph
      Nov 12 at 19:23










    • @joseph It really doesn't matter, as already written there. You can do it correctly. The important point is already there...
      – DonAntonio
      Nov 12 at 19:33
















    Check carefully, in case you want/need to do it, the signs of the simple fractions as I have some minor doubts about that...but that really is irrelevant for the direct question you've asked.
    – DonAntonio
    Nov 12 at 17:44




    Check carefully, in case you want/need to do it, the signs of the simple fractions as I have some minor doubts about that...but that really is irrelevant for the direct question you've asked.
    – DonAntonio
    Nov 12 at 17:44












    In the first summation term, why is it $(x - 1)^{2}$ rather than $(x - 1)^{n}$?
    – joseph
    Nov 12 at 18:05




    In the first summation term, why is it $(x - 1)^{2}$ rather than $(x - 1)^{n}$?
    – joseph
    Nov 12 at 18:05












    @joseph Simple typo. Corrected.
    – DonAntonio
    Nov 12 at 18:18




    @joseph Simple typo. Corrected.
    – DonAntonio
    Nov 12 at 18:18












    I think your partial fraction decomposition in the first equation is incorrect. The coefficient of $frac{1}{4}$ should be $+$ not $-$.
    – joseph
    Nov 12 at 19:23




    I think your partial fraction decomposition in the first equation is incorrect. The coefficient of $frac{1}{4}$ should be $+$ not $-$.
    – joseph
    Nov 12 at 19:23












    @joseph It really doesn't matter, as already written there. You can do it correctly. The important point is already there...
    – DonAntonio
    Nov 12 at 19:33




    @joseph It really doesn't matter, as already written there. You can do it correctly. The important point is already there...
    – DonAntonio
    Nov 12 at 19:33










    up vote
    1
    down vote













    If you represent $frac {1}{(3-x)(1+x)^2}$ as a Taylor series, centered at the points identified above that Taylor series will have a finite radius. There is enough information presented in the question to find these radii without deriving the entire series.



    However, if you represent the function as a Laurent series, you have more choices in how you represent the series and depending on those choices the series will converge in different annuli.



    The trick comes down to finding the discontinuities of the function. Since the Taylor series is continuous, it is going to diverge as these discontinuities approach.






    share|cite|improve this answer























    • is this a good way to do it? the taylor series representation of the function looks very ugly wolframalpha.com/input/…
      – joseph
      Nov 12 at 17:33












    • If you work out the Taylor series centered at $0,$ you are going to get something of the form $sum A (frac {x}{3})^n + B (-1)^nx^n + Cn(-1)^n x^n$
      – Doug M
      Nov 12 at 17:47

















    up vote
    1
    down vote













    If you represent $frac {1}{(3-x)(1+x)^2}$ as a Taylor series, centered at the points identified above that Taylor series will have a finite radius. There is enough information presented in the question to find these radii without deriving the entire series.



    However, if you represent the function as a Laurent series, you have more choices in how you represent the series and depending on those choices the series will converge in different annuli.



    The trick comes down to finding the discontinuities of the function. Since the Taylor series is continuous, it is going to diverge as these discontinuities approach.






    share|cite|improve this answer























    • is this a good way to do it? the taylor series representation of the function looks very ugly wolframalpha.com/input/…
      – joseph
      Nov 12 at 17:33












    • If you work out the Taylor series centered at $0,$ you are going to get something of the form $sum A (frac {x}{3})^n + B (-1)^nx^n + Cn(-1)^n x^n$
      – Doug M
      Nov 12 at 17:47















    up vote
    1
    down vote










    up vote
    1
    down vote









    If you represent $frac {1}{(3-x)(1+x)^2}$ as a Taylor series, centered at the points identified above that Taylor series will have a finite radius. There is enough information presented in the question to find these radii without deriving the entire series.



    However, if you represent the function as a Laurent series, you have more choices in how you represent the series and depending on those choices the series will converge in different annuli.



    The trick comes down to finding the discontinuities of the function. Since the Taylor series is continuous, it is going to diverge as these discontinuities approach.






    share|cite|improve this answer














    If you represent $frac {1}{(3-x)(1+x)^2}$ as a Taylor series, centered at the points identified above that Taylor series will have a finite radius. There is enough information presented in the question to find these radii without deriving the entire series.



    However, if you represent the function as a Laurent series, you have more choices in how you represent the series and depending on those choices the series will converge in different annuli.



    The trick comes down to finding the discontinuities of the function. Since the Taylor series is continuous, it is going to diverge as these discontinuities approach.







    share|cite|improve this answer














    share|cite|improve this answer



    share|cite|improve this answer








    edited Nov 12 at 17:35

























    answered Nov 12 at 17:33









    Doug M

    42.5k31752




    42.5k31752












    • is this a good way to do it? the taylor series representation of the function looks very ugly wolframalpha.com/input/…
      – joseph
      Nov 12 at 17:33












    • If you work out the Taylor series centered at $0,$ you are going to get something of the form $sum A (frac {x}{3})^n + B (-1)^nx^n + Cn(-1)^n x^n$
      – Doug M
      Nov 12 at 17:47




















    • is this a good way to do it? the taylor series representation of the function looks very ugly wolframalpha.com/input/…
      – joseph
      Nov 12 at 17:33












    • If you work out the Taylor series centered at $0,$ you are going to get something of the form $sum A (frac {x}{3})^n + B (-1)^nx^n + Cn(-1)^n x^n$
      – Doug M
      Nov 12 at 17:47


















    is this a good way to do it? the taylor series representation of the function looks very ugly wolframalpha.com/input/…
    – joseph
    Nov 12 at 17:33






    is this a good way to do it? the taylor series representation of the function looks very ugly wolframalpha.com/input/…
    – joseph
    Nov 12 at 17:33














    If you work out the Taylor series centered at $0,$ you are going to get something of the form $sum A (frac {x}{3})^n + B (-1)^nx^n + Cn(-1)^n x^n$
    – Doug M
    Nov 12 at 17:47






    If you work out the Taylor series centered at $0,$ you are going to get something of the form $sum A (frac {x}{3})^n + B (-1)^nx^n + Cn(-1)^n x^n$
    – Doug M
    Nov 12 at 17:47












    up vote
    1
    down vote













    Here are my thoughts. This might not be the original purpose of the exercise, but this is my guess.



    According to your partial fraction decomposition, and using the well-know power series $(1-x)^{-1} = sum_0^infty x^n$ for $x in (-1,1)$, at $x=0$, we have
    begin{align*}
    f(x)&= frac 1{16}sum_0^infty (-1)^n x^n - frac {mathrm d}{mathrm dx} left( frac 1{4(x+1)} right) + frac 1{48} cdot frac 1{1 - x/3} \
    &= frac 1{16}sum_0^infty (-1)^n x^n - frac 14 cdotfrac {mathrm d }{mathrm dx}left( sum_0^infty (-1)^n x^n right) + frac 1{48} cdot sum_0^infty frac {x^n}{3^n}\
    &= frac 1{16}sum_0^infty (-1)^n x^n - frac 14 sum_0^infty (n+1)(-1)^{n+1} x^{n} + frac 1{48} cdot sum_0^infty frac {x^n}{3^n}\
    &= sum_0^infty left(frac {(-1)^n}{16} + frac 14 (n+1)(-1)^{n+1} +frac 1{16 cdot 3^{n+1}} right) x^n,
    end{align*}

    and all of these computations are valid iff $|x|<1$ and $|x/3|<1$. Conclusively the radius is $1$. You might try to compute again by the formula.



    At $x=1$, just let $y = x-1$, then $x = y+1$ and
    $$
    f(x) =g(y) = frac 1{(2-y)(2+y)^2},
    $$

    you could do the similar thing like above to give the power series at $y=0$. You could see that the operations are valid iff $|y/2|<1$, hence $|y|<2$ and the radius is $2$.






    share|cite|improve this answer

























      up vote
      1
      down vote













      Here are my thoughts. This might not be the original purpose of the exercise, but this is my guess.



      According to your partial fraction decomposition, and using the well-know power series $(1-x)^{-1} = sum_0^infty x^n$ for $x in (-1,1)$, at $x=0$, we have
      begin{align*}
      f(x)&= frac 1{16}sum_0^infty (-1)^n x^n - frac {mathrm d}{mathrm dx} left( frac 1{4(x+1)} right) + frac 1{48} cdot frac 1{1 - x/3} \
      &= frac 1{16}sum_0^infty (-1)^n x^n - frac 14 cdotfrac {mathrm d }{mathrm dx}left( sum_0^infty (-1)^n x^n right) + frac 1{48} cdot sum_0^infty frac {x^n}{3^n}\
      &= frac 1{16}sum_0^infty (-1)^n x^n - frac 14 sum_0^infty (n+1)(-1)^{n+1} x^{n} + frac 1{48} cdot sum_0^infty frac {x^n}{3^n}\
      &= sum_0^infty left(frac {(-1)^n}{16} + frac 14 (n+1)(-1)^{n+1} +frac 1{16 cdot 3^{n+1}} right) x^n,
      end{align*}

      and all of these computations are valid iff $|x|<1$ and $|x/3|<1$. Conclusively the radius is $1$. You might try to compute again by the formula.



      At $x=1$, just let $y = x-1$, then $x = y+1$ and
      $$
      f(x) =g(y) = frac 1{(2-y)(2+y)^2},
      $$

      you could do the similar thing like above to give the power series at $y=0$. You could see that the operations are valid iff $|y/2|<1$, hence $|y|<2$ and the radius is $2$.






      share|cite|improve this answer























        up vote
        1
        down vote










        up vote
        1
        down vote









        Here are my thoughts. This might not be the original purpose of the exercise, but this is my guess.



        According to your partial fraction decomposition, and using the well-know power series $(1-x)^{-1} = sum_0^infty x^n$ for $x in (-1,1)$, at $x=0$, we have
        begin{align*}
        f(x)&= frac 1{16}sum_0^infty (-1)^n x^n - frac {mathrm d}{mathrm dx} left( frac 1{4(x+1)} right) + frac 1{48} cdot frac 1{1 - x/3} \
        &= frac 1{16}sum_0^infty (-1)^n x^n - frac 14 cdotfrac {mathrm d }{mathrm dx}left( sum_0^infty (-1)^n x^n right) + frac 1{48} cdot sum_0^infty frac {x^n}{3^n}\
        &= frac 1{16}sum_0^infty (-1)^n x^n - frac 14 sum_0^infty (n+1)(-1)^{n+1} x^{n} + frac 1{48} cdot sum_0^infty frac {x^n}{3^n}\
        &= sum_0^infty left(frac {(-1)^n}{16} + frac 14 (n+1)(-1)^{n+1} +frac 1{16 cdot 3^{n+1}} right) x^n,
        end{align*}

        and all of these computations are valid iff $|x|<1$ and $|x/3|<1$. Conclusively the radius is $1$. You might try to compute again by the formula.



        At $x=1$, just let $y = x-1$, then $x = y+1$ and
        $$
        f(x) =g(y) = frac 1{(2-y)(2+y)^2},
        $$

        you could do the similar thing like above to give the power series at $y=0$. You could see that the operations are valid iff $|y/2|<1$, hence $|y|<2$ and the radius is $2$.






        share|cite|improve this answer












        Here are my thoughts. This might not be the original purpose of the exercise, but this is my guess.



        According to your partial fraction decomposition, and using the well-know power series $(1-x)^{-1} = sum_0^infty x^n$ for $x in (-1,1)$, at $x=0$, we have
        begin{align*}
        f(x)&= frac 1{16}sum_0^infty (-1)^n x^n - frac {mathrm d}{mathrm dx} left( frac 1{4(x+1)} right) + frac 1{48} cdot frac 1{1 - x/3} \
        &= frac 1{16}sum_0^infty (-1)^n x^n - frac 14 cdotfrac {mathrm d }{mathrm dx}left( sum_0^infty (-1)^n x^n right) + frac 1{48} cdot sum_0^infty frac {x^n}{3^n}\
        &= frac 1{16}sum_0^infty (-1)^n x^n - frac 14 sum_0^infty (n+1)(-1)^{n+1} x^{n} + frac 1{48} cdot sum_0^infty frac {x^n}{3^n}\
        &= sum_0^infty left(frac {(-1)^n}{16} + frac 14 (n+1)(-1)^{n+1} +frac 1{16 cdot 3^{n+1}} right) x^n,
        end{align*}

        and all of these computations are valid iff $|x|<1$ and $|x/3|<1$. Conclusively the radius is $1$. You might try to compute again by the formula.



        At $x=1$, just let $y = x-1$, then $x = y+1$ and
        $$
        f(x) =g(y) = frac 1{(2-y)(2+y)^2},
        $$

        you could do the similar thing like above to give the power series at $y=0$. You could see that the operations are valid iff $|y/2|<1$, hence $|y|<2$ and the radius is $2$.







        share|cite|improve this answer












        share|cite|improve this answer



        share|cite|improve this answer










        answered Nov 12 at 17:44









        xbh

        4,7941421




        4,7941421






























             

            draft saved


            draft discarded



















































             


            draft saved


            draft discarded














            StackExchange.ready(
            function () {
            StackExchange.openid.initPostLogin('.new-post-login', 'https%3a%2f%2fmath.stackexchange.com%2fquestions%2f2995568%2ffinding-the-radius-of-convergence-for-frac13-x1-x2-at-x-0%23new-answer', 'question_page');
            }
            );

            Post as a guest















            Required, but never shown





















































            Required, but never shown














            Required, but never shown












            Required, but never shown







            Required, but never shown

































            Required, but never shown














            Required, but never shown












            Required, but never shown







            Required, but never shown







            Popular posts from this blog

            Biblatex bibliography style without URLs when DOI exists (in Overleaf with Zotero bibliography)

            ComboBox Display Member on multiple fields

            Is it possible to collect Nectar points via Trainline?